Themenbereiche Themenbereiche Profile Hilfe/Anleitungen Help    
Recent Posts Last 1|3|7 Days Suche Suche Tree Tree View  

Brüche

ZahlReich - Mathematik Hausaufgabenhilfe » ---- Archiv: Klasse 11 » Folgen und Reihen » Brüche « Zurück Vor »

Autor Beitrag
Seitenanfangvoriger Beitragnächster BeitragSeitenende Link zu diesem Beitrag

missi (missi)
Suche alle Beiträge dieser Person in dieser Hauptrubrik
Neues Mitglied
Benutzername: missi

Nummer des Beitrags: 2
Registriert: 03-2002
Veröffentlicht am Samstag, den 27. Juli, 2002 - 11:25:   Beitrag drucken

Wie wandelt man bitte einen unendlichen, periodischen Dezimalbruch in einen gemeinen Bruch um?
Seitenanfangvoriger Beitragnächster BeitragSeitenende Link zu diesem Beitrag

Josef Filipiak (filipiak)
Suche alle Beiträge dieser Person in dieser Hauptrubrik
Neues Mitglied
Benutzername: filipiak

Nummer des Beitrags: 2
Registriert: 10-2001
Veröffentlicht am Samstag, den 27. Juli, 2002 - 12:12:   Beitrag drucken

Hallo missi, zur Umwandlung eines periodischen Dezimalbruchs machst du dir die Periode an sich nutzbar. Die Dezimalzahl wird mit einer Zehnerpotenz multipliziert, die der Länge der Periode entspricht. Anschließend wird von diesem Vielfachen der Dezimalbruch abgezogen. Dabei verschwindet die Periode. Diese ergibt den Zähler des echten Bruchs. Den Nenner erhältst du, indem von der Zehnerpotenz 1 abgezogen wird. Anschließend kannst du wie üblich kürzen:

Beispiel:
Dezimalbruch: 0,675675... Periode = 675 Multiplikation mit 1000
Rechnung: 675,675 - 0,675 = 675 = Zähler
1000 - 1 = 999 = Nenner
Ergebnis: 0,675 = 675/999 = 75/111

Dies funktioniert auch mit gemischt-periodischen Brüchen:
Dezimalbruch: 0,532121.. Periode = 21 Multiplikation mit 100
Rechnung: 53,2121 - 0,5321 = 52,68 = Zähler
100 - 1 = Nenner.
Um kein Komma im Bruch zu schreiben, wird mit 100 erweitert und anschließend mit 12 gekürzt:
Ergebnis: 52.68/99 * 100/100 = 5268/9900 = 439/825.

Gruß Filipiak
Seitenanfangvoriger Beitragnächster BeitragSeitenende Link zu diesem Beitrag

missi (missi)
Suche alle Beiträge dieser Person in dieser Hauptrubrik
Neues Mitglied
Benutzername: missi

Nummer des Beitrags: 3
Registriert: 03-2002
Veröffentlicht am Samstag, den 27. Juli, 2002 - 16:33:   Beitrag drucken

Vielen Dank. Kannst du mir auch sagen, wieso das beim Thema "Reihen" behandelt wird? Braucht man das dafür?
Seitenanfangvoriger Beitragnächster BeitragSeitenende Link zu diesem Beitrag

Christian Schmidt (christian_s)
Suche alle Beiträge dieser Person in dieser Hauptrubrik
Mitglied
Benutzername: christian_s

Nummer des Beitrags: 16
Registriert: 02-2002
Veröffentlicht am Samstag, den 27. Juli, 2002 - 17:24:   Beitrag drucken

Hi missi

Ohne mir den Beitrag von Filipiak genauer durchzulesen, würde ich sagen, dass das etwas mit der geometrischen Reihe zu tun hat. Hast du beispielsweise den unendlichen, periodischen Dezimalbruch
0,99999.....
also Bruch geschrieben ist dieser laut der Regel von Filipiak 9/9=1. Den "Beweis" dafür erhältst du durch die Summenformel für die geometrische Reihe:
Sn i=0 q^i=(1-q^(n+1))/(1-q)

Unser Bruch als unendlich Reihe geschrieben hätte folgende Darstellung:
9*(Sn i=0 (1/10)^i )-9=9*((1-(1/10)^(n+1))/(1-q))-9
Wir müssen jetzt nur noch den Grenzwert n->oo betrachten. Wie man leicht sieht verschwindet der Term (1/10)^(n+1) und man erhält:
lim(n->oo) 9*((1-(1/10)^(n+1))/(1-q))-9=9*10/9-9=1
Also 0,9999....=9/9=1

Ich hoffe mal ich hab mich nicht verrechnet. Wenn du das noch ein bißchen verallgemeinerst, erhältst du sicher die allgemeine Regel von Filipiak.

MfG
C. Schmidt
Seitenanfangvoriger Beitragnächster BeitragSeitenende Link zu diesem Beitrag

missi (missi)
Suche alle Beiträge dieser Person in dieser Hauptrubrik
Junior Mitglied
Benutzername: missi

Nummer des Beitrags: 9
Registriert: 03-2002
Veröffentlicht am Mittwoch, den 31. Juli, 2002 - 14:13:   Beitrag drucken

Vielen Dank für deine Mühe, aber 1 ist doch nicht dasselbe wie 0,99999...?!
Seitenanfangvoriger Beitragnächster BeitragSeitenende Link zu diesem Beitrag

Robert (emperor2002)
Suche alle Beiträge dieser Person in dieser Hauptrubrik
Mitglied
Benutzername: emperor2002

Nummer des Beitrags: 21
Registriert: 04-2002
Veröffentlicht am Mittwoch, den 31. Juli, 2002 - 14:20:   Beitrag drucken

Hi missi!

Mal schnell einen Beweis, dass 1 = 0.p9 ist:

Wir gehen einfach mal davon aus das es so ist:

I: 1 = 0.p9 | · 10
II: 10 = 9.p9

II - I => 9 = 9 w.A

Da wir eine wahre Aussage erhalten, muss die Behauptung dass 1 = 0.p9 ist wahr sein!

:-)
MFG Robert

www.mathefreak.de / webmaster@mathefreak.de
Seitenanfangvoriger Beitragnächster BeitragSeitenende Link zu diesem Beitrag

Christian Schmidt (christian_s)
Suche alle Beiträge dieser Person in dieser Hauptrubrik
Mitglied
Benutzername: christian_s

Nummer des Beitrags: 31
Registriert: 02-2002
Veröffentlicht am Mittwoch, den 31. Juli, 2002 - 14:26:   Beitrag drucken

Hi Robert

Man kann aber doch auch aus Falschem Wahres folgern.

MfG
C. Schmidt
Seitenanfangvoriger Beitragnächster BeitragSeitenende Link zu diesem Beitrag

Robert (emperor2002)
Suche alle Beiträge dieser Person in dieser Hauptrubrik
Mitglied
Benutzername: emperor2002

Nummer des Beitrags: 22
Registriert: 04-2002
Veröffentlicht am Mittwoch, den 31. Juli, 2002 - 14:35:   Beitrag drucken

@Christian

Sowas würde ich dann Widerspruchsbeweis nennen *g*
Natürlich hast du aber recht, was aber nix am Beweis ändert, oder willst du auf konkretes hinaus?

Frosch
MFG Robert

www.mathefreak.de / webmaster@mathefreak.de
Seitenanfangvoriger Beitragnächster BeitragSeitenende Link zu diesem Beitrag

Christian Schmidt (christian_s)
Suche alle Beiträge dieser Person in dieser Hauptrubrik
Mitglied
Benutzername: christian_s

Nummer des Beitrags: 32
Registriert: 02-2002
Veröffentlicht am Mittwoch, den 31. Juli, 2002 - 14:55:   Beitrag drucken

Hi Ropert

Ich hab eigentlich nicht besonders viel Ahnung von Logik....
Ich meinte nur:
Du setzt das zu beweisende als wahr voraus und folgerst daraus eine wahre Aussage und schließt dann darauf, dass die zu beweisende Aussage ´wahr ist.
ist A: 1 = 0.p9
B: 9 = 9

Bei A steht es ja jetzt noch nicht fest, ob die Aussage stimmt, also ist A wahr oder falsch.
A=>B
wobei B wahr ist.
Also kann A wahr oder falsch sein, wir sind also wieder am Anfang.(Ich hab keine Ahnung, ob meine Argumentation hier so überhaupt stimmt, aber an der Uni hab ich mal für so nen ähnlichen Beweis nen Fehler bekommen.)

Jetzt nochmal zum Widerspruchsbeweis.
Hier nimmt man an, dass eine Aussage A wahr ist und folgert daraus eine falsche Aussage B. Da man aus wahrem nichts falsches folgern kann, muss Aussage A falsch sein.

MfG
C. Schmidt
Seitenanfangvoriger Beitragnächster BeitragSeitenende Link zu diesem Beitrag

Robert (emperor2002)
Suche alle Beiträge dieser Person in dieser Hauptrubrik
Mitglied
Benutzername: emperor2002

Nummer des Beitrags: 23
Registriert: 04-2002
Veröffentlicht am Mittwoch, den 31. Juli, 2002 - 15:12:   Beitrag drucken

@christian

Nehmen wir mal an 1 = 0.p9 wäre nicht wahr. Dann düfte man durch Termumformung irgendwann einmal auf einen Widerspruch treffen (falsche Aussage würde entstehen).

Ich habe also vorausgesetzt das 1 = 0.p9 wahr ist. Durch Umformung komme ich auf eine wahre aussage => Die Ausgangsgleichung ist auch wahr.

Nehmen wir mal ein Beispiel wo wir was flasches Voraussetzen:

10 = 9 + 2 (Wir setzen das als "wahr" voraus)
8 = 9 f.A => somit muss auch unsere Ausgangsgleichung falsch sein, da wir nur äquivalente Umformungen durchgeführt haben!

Ich weis das dieses Beispiel nun wirklich zum Lachen ist, aber (meiner Meinung und auch meines Wissens (is ja erst Stand 11.Klasse)) wenn ich von etwas ausgehe und ich es als richtig anerkenne, und ich erhalte durch Umformungen eine falsche Aussage, so muss auch die Ausgangsgleichung falsch sein! Das klingt für mich Logisch :-)

Es ist ja mal ganz grob betrachtet ähnlich der voll. Induktion, man geht von etwas aus, wovon man nicht weiß ob es wahr oder falsch ist.
Das soll jetzt nicht bedeuten das der Beweis eine voll.Ind. ist, das ist mir auch klar, es geht mir nur im den Sinn!

:-)


MFG Robert

www.mathefreak.de / webmaster@mathefreak.de
Seitenanfangvoriger Beitragnächster BeitragSeitenende Link zu diesem Beitrag

Christian Schmidt (christian_s)
Suche alle Beiträge dieser Person in dieser Hauptrubrik
Mitglied
Benutzername: christian_s

Nummer des Beitrags: 33
Registriert: 02-2002
Veröffentlicht am Mittwoch, den 31. Juli, 2002 - 15:25:   Beitrag drucken

Hi Robert

10 = 9 + 2 (Wir setzen das als "wahr" voraus)
8 = 9 f.A => somit muss auch unsere Ausgangsgleichung falsch sein, da wir nur äquivalente Umformungen durchgeführt haben!


Das ist jetzt ein Widerspruchsbeweis und der stimmt auf jeden Fall, weil du etwas falsches folgerst und das ist unmöglich. (Aus wahrem kann man nichts falsches folgern)

Mir fällt kein Beispiel ein, wo man aus einer richtigen Gleichung eine falsche folgert. Vielleicht gibt es das auch gar nicht. Aber bei anderen Sachen geht das auf jeden Fall. Beispiele gibts hier:

http://www.mathematik.net/logik/11k2s6.htm

Ich weis das dieses Beispiel nun wirklich zum Lachen ist, aber (meiner Meinung und auch meines Wissens (is ja erst Stand 11.Klasse)) wenn ich von etwas ausgehe und ich es als richtig anerkenne, und ich erhalte durch Umformungen eine falsche Aussage, so muss auch die Ausgangsgleichung falsch sein! Das klingt für mich Logisch

Das stimmt ja auch. Nur wenn du etwas wahres folgerst kann halt wie gesagt die Prämisse wahr oder falsch sein und man hat gar nichts bewiesen.

Es ist ja mal ganz grob betrachtet ähnlich der voll. Induktion, man geht von etwas aus, wovon man nicht weiß ob es wahr oder falsch ist.

Deshalb macht man ja auch den Induktionsanfang. Dabei ist es mir übrigens mal passiert, dass ich den weggelassen hab und bei der Induktion kam am Ende was vernünftiges raus. Nur leider gab es halt keine Startwert für den die Behauptung galt...;)

MfG
C. Schmidt

Seitenanfangvoriger Beitragnächster BeitragSeitenende Link zu diesem Beitrag

Robert (emperor2002)
Suche alle Beiträge dieser Person in dieser Hauptrubrik
Mitglied
Benutzername: emperor2002

Nummer des Beitrags: 24
Registriert: 04-2002
Veröffentlicht am Mittwoch, den 31. Juli, 2002 - 15:38:   Beitrag drucken

@Christian

Gut lassen wir es dabei! Ich werde mich mal irgendwo genauer schlau machen, ob der Beweis gültig ist!

:-)
Weils hier so warm ist: ventilator
MFG Robert

www.mathefreak.de / webmaster@mathefreak.de
Seitenanfangvoriger Beitragnächster BeitragSeitenende Link zu diesem Beitrag

Christian Schmidt (christian_s)
Suche alle Beiträge dieser Person in dieser Hauptrubrik
Mitglied
Benutzername: christian_s

Nummer des Beitrags: 35
Registriert: 02-2002
Veröffentlicht am Mittwoch, den 31. Juli, 2002 - 15:43:   Beitrag drucken

Jo, ist wohl besser so :-)

Aber meine ganzen Aussagen sind ja eher Fragen, ich bin mir ja selbst nicht sicher;)
Interesseieren würds micht trotzdem. Vielleicht schaut sich das ja mal irgendjemand anderes an.

MfG
C. Schmidt

PS:Danke für die Abkühlung :-)
Seitenanfangvoriger Beitragnächster BeitragSeitenende Link zu diesem Beitrag

Zaph (zaph)
Suche alle Beiträge dieser Person in dieser Hauptrubrik
Senior Mitglied
Benutzername: zaph

Nummer des Beitrags: 1257
Registriert: 07-2000
Veröffentlicht am Mittwoch, den 31. Juli, 2002 - 21:58:   Beitrag drucken

Nehmen wir mal an, dass wir beweisen wollen

10 = 9 + 2

=> (beide Seiten minus 10,5)

-0,5 = 0,5

=> (beide Seiten quadrieren)

0,25 = 0,25

Das ist wahr, also 10 = 9 + 2.

q. e. d. und Gruß

Z.
Seitenanfangvoriger Beitragnächster BeitragSeitenende Link zu diesem Beitrag

Robert (emperor2002)
Suche alle Beiträge dieser Person in dieser Hauptrubrik
Mitglied
Benutzername: emperor2002

Nummer des Beitrags: 25
Registriert: 04-2002
Veröffentlicht am Mittwoch, den 31. Juli, 2002 - 22:41:   Beitrag drucken

@Zaph

Das ist wohl war, aber dir dürfte eigentlich beim Lesen auch aufgefallen sein, das ich von "äquivalenten" Umformungsschritten gesprochen habe! Und Quadrieren fällt in diese Kategorie nicht hinein!

Gruß Robert
MFG Robert

www.mathefreak.de / webmaster@mathefreak.de
Seitenanfangvoriger Beitragnächster BeitragSeitenende Link zu diesem Beitrag

Zaph (zaph)
Suche alle Beiträge dieser Person in dieser Hauptrubrik
Senior Mitglied
Benutzername: zaph

Nummer des Beitrags: 1258
Registriert: 07-2000
Veröffentlicht am Donnerstag, den 01. August, 2002 - 00:17:   Beitrag drucken

Nein, so ganz klar war mir das nicht.

Zeichen für Folgerung: =>

Zeichen für äquivalente Umformung: <=>

Und deine Folgerung im Beitrag von 31. Juli, 15:20 war bestimmt keine äquivalente Umformung.

Gruß

Z.
Seitenanfangvoriger Beitragnächster BeitragSeitenende Link zu diesem Beitrag

Robert (emperor2002)
Suche alle Beiträge dieser Person in dieser Hauptrubrik
Mitglied
Benutzername: emperor2002

Nummer des Beitrags: 26
Registriert: 04-2002
Veröffentlicht am Donnerstag, den 01. August, 2002 - 00:24:   Beitrag drucken

@Zaph

Gut ich habe das Symbol "=>" nicht dazu verwendet um äquivalenz oder Folgerung auszudrücken. Ich nehms halt gerne für Schlussfolgerungen. Aber:

"10 = 9 + 2 (Wir setzen das als "wahr" voraus)
8 = 9 f.A => somit muss auch unsere Ausgangsgleichung falsch sein, da wir nur äquivalente Umformungen durchgeführt haben!"

Ich habs versucht zu verdeutlichen. Naja egal, es bringt nichts über geschriebenes zu diskutieren.

Wie siehst du denn den Beweis. Ist er dir plausibel oder bemängelst du auch das was Christian bemängelt

:-)
PS: Es ist immer noch verdammt heiß hier *gg*

MFG Robert

www.mathefreak.de / webmaster@mathefreak.de
Seitenanfangvoriger Beitragnächster BeitragSeitenende Link zu diesem Beitrag

clara
Suche alle Beiträge dieser Person in dieser Hauptrubrik
Unregistrierter Gast
Veröffentlicht am Donnerstag, den 01. August, 2002 - 16:05:   Beitrag drucken

Hi zusammen,
noch mal zu 1=0.p9.
Wahr ist:
1/3=0,p3 und damit ist:
1 = 3*1/3 = 3*0,p3 = 0,p9

Gruß clara
Seitenanfangvoriger Beitragnächster BeitragSeitenende Link zu diesem Beitrag

Zaph (zaph)
Suche alle Beiträge dieser Person in dieser Hauptrubrik
Senior Mitglied
Benutzername: zaph

Nummer des Beitrags: 1262
Registriert: 07-2000
Veröffentlicht am Donnerstag, den 01. August, 2002 - 17:44:   Beitrag drucken

@clara: Wieso so einfach? ;-)

@Robert: Wenn A und B Aussagen sind und du die Folgerung A => B beweisen kannst und du außerdem weißt, dass B falsch ist, DANN ist auch A falsch.

Aber, wenn du weißt, dass A => B gilt und B wahr ist, dann weißt du über A rein überhaupt nichts!

In deinem Beitrag vom 31. Juli - 15:20 ist

Aussage A) a = b und c = d
Aussage B) a - c = b - d

(mit a = 1, b = 0,p9, c = 10, d = 9,p9)

Es stimmt zwar, dass "A => B" und B wahr sind, aber daraus lässt sich nichts über den Wahrheitsgehalt von A sagen.
Seitenanfangvoriger Beitragnächster BeitragSeitenende Link zu diesem Beitrag

Robert (emperor2002)
Suche alle Beiträge dieser Person in dieser Hauptrubrik
Mitglied
Benutzername: emperor2002

Nummer des Beitrags: 28
Registriert: 04-2002
Veröffentlicht am Donnerstag, den 01. August, 2002 - 18:02:   Beitrag drucken

@Zaph!

Gut danke! Ich habs ja eingesehen.

Nur: Aussage B ist bei mir durch Umformung von A entstanden:

I: 1 = 0.p9 |·10
I': 10 = 9.p9
----------------
i' - I: 9 = 9

Irgendwie habe ich ja keine 2 Behauptungen aufgestellt. Ich habe nur angenommen das A war ist, und A durch äquivalente Umformungen verändert, und dabei eine wahre Aussage erhalten. Und deshalb scheint mir hier logisch, dass auch die Anfangsgleichung wahr ist!!

Also wie schon im Vorposting "=>" sollte nicht heißen "aus A folgt B"!

Gruß
MFG Robert

www.mathefreak.de / webmaster@mathefreak.de
Seitenanfangvoriger Beitragnächster BeitragSeitenende Link zu diesem Beitrag

Zaph (zaph)
Suche alle Beiträge dieser Person in dieser Hauptrubrik
Senior Mitglied
Benutzername: zaph

Nummer des Beitrags: 1263
Registriert: 07-2000
Veröffentlicht am Donnerstag, den 01. August, 2002 - 18:31:   Beitrag drucken

Bitte sieh es doch ein: Die Umformung ist NICHT äquivalent!!!

Was soll ich denn noch sagen? Hast du mein letztes Posting überhaupt richtig gelesen? (Beachte bitte, dass dein "A" nicht mit meinem "A" übereinstimmt.)
Seitenanfangvoriger Beitragnächster BeitragSeitenende Link zu diesem Beitrag

Robert (emperor2002)
Suche alle Beiträge dieser Person in dieser Hauptrubrik
Mitglied
Benutzername: emperor2002

Nummer des Beitrags: 29
Registriert: 04-2002
Veröffentlicht am Donnerstag, den 01. August, 2002 - 18:43:   Beitrag drucken

@Zaph!

Ich sehe es ja ein was du im Posting vom 01. August, 2002 - 18:44 geschrieben hast, dass man bloß weil B wahr ist nicht auf A schließen kann.

Nur versuche ich die ganze Zeit herauszufinden wo bei der Rechnung:

I: 1 = 0.p9 |·10
I': 10 = 9.p9
----------------
i' - I: 9 = 9

keine äquivalente Umformung ist. Ich habe nur multipliziert und subtrahiert, was nach meinem Kenntnisstand äquivalente Umformungen sind.

Ich will jetz hier auch nicht weiter darüber rumdiskutieren, aber mir gehts nur darum, das du mir zeigst wo in dieser Rechnung KEINE äqu.Umformungen sind!!!


MFG Robert

www.mathefreak.de / webmaster@mathefreak.de
Seitenanfangvoriger Beitragnächster BeitragSeitenende Link zu diesem Beitrag

Jan Martin Krämer (species5672)
Suche alle Beiträge dieser Person in dieser Hauptrubrik
Mitglied
Benutzername: species5672

Nummer des Beitrags: 41
Registriert: 07-2002
Veröffentlicht am Donnerstag, den 01. August, 2002 - 21:04:   Beitrag drucken

@Robert

Du darfst nicht !!! etwas als wahr annehmen, damit rumrechnen und aus der Tatsache das dann etwas wahres rauskommt behaupten das deine Annahme stimmt.

Das wäre ein Beweis der folgenden Art:

Annahme: Einstein war ein Idiot.

Nach unserer Annahme war Einstein ein Idiot was nach unserer Annahme war ist, also ist Einstein ein Idiot.
qed

Das ganze nochmal mathematisch und exakt nach deinem Beweisschema:

Annahme: 10 = 9 + 2
I: 10 = 9 + 2
II: (10) + 3 = (9 + 2) + 3
I-II: -3 = -3

Es ergibt sich eine wahre Aussage, also ist 10 = 9 + 2

Ich verstehe ehrlich gesagt nicht ganz was Zaph mit der nicht äquivalenten Aussage meinte, was du falsch gemacht hast ist folgendes:

Du hast angenommen, dass 1=0,999..
Daraus hast du geschlossen, dass 10=9,999.. (korrekt, aber nur FALLS!!! 1=0,999...)
Das hast du dann umgeformt in 10-1 = 9,999.. - 0,999...
was äquivalent zu 9=9 ist.
Das gilt aber nur und wirklich auch nur FALLS!!!: 1=0,999.. ist, was du aber gerade beweisen wolltest!
Damit hast du gezeigt, das man aus 1=0,999.. schließen kann, das 9=9 ist und das 10=9,999.. was aber nach Zaphs Erläuterung NICHTS aber auch GAR NICHTS über den Wahrheitsgehalt der Aussage 1=0,999.. aussagt.

Noch einmal deutlich:

Mit der Behauptung A=>B kann man nichts anfangen, außer B ist falsch, dann ist auch A falsch.
Also darf man nicht annehmen das etwas wahr ist, um tatsächlich zu zeigen das es wahr ist!
Wenn man annimmt das etwas wahr ist, dann kann man nur zeigen das es falsch ist!
Seitenanfangvoriger Beitragnächster BeitragSeitenende Link zu diesem Beitrag

Robert (emperor2002)
Suche alle Beiträge dieser Person in dieser Hauptrubrik
Mitglied
Benutzername: emperor2002

Nummer des Beitrags: 30
Registriert: 04-2002
Veröffentlicht am Donnerstag, den 01. August, 2002 - 21:15:   Beitrag drucken

@Jan

Gut! Jetz blicke ich durch! Danke für dein Beispiel. Überzeugt bin ich jetzt auch nur tut ein Gegenbeispiel immer gut!

THX @Zaph & Jan
MFG Robert

www.mathefreak.de / webmaster@mathefreak.de
Seitenanfangvoriger Beitragnächster BeitragSeitenende Link zu diesem Beitrag

Walter H. (mainziman)
Suche alle Beiträge dieser Person in dieser Hauptrubrik
Fortgeschrittenes Mitglied
Benutzername: mainziman

Nummer des Beitrags: 99
Registriert: 05-2002
Veröffentlicht am Donnerstag, den 01. August, 2002 - 21:20:   Beitrag drucken

Hi,

hier mal ein Verfahren, welches periodische Dezimalzahlen in einen Bruch verwandelt:

unsere periodische Zahl lautet 0,p9

x = 0,p9
<=>
10*x = 9,p9
beide äquivalenten Gleichungen subtrahieren
9*x = 9
<=>
x = 1

eine andere periodische Zahl
0,001001001001001001001001001 ...

x = 0,001001001001001 ...
<=>
1000*x = 1,001001001001 ...
beide äquivalenten Gleichungen subtrahieren
999*x = 1
<=>
x = 1/999

Alles klar, wie dieses Verfahren funktioniert?

Gruß,
Walter

p.s. 0,99999999 ist eins, man kann es auch anders herum zeigen

1 - 0,p9 = lim[x->+inf] 1/x meine Annahme
1 - 0,p9 = 0
=> 0,p9 muß 1 sein

Mainzi Man,
ein Mainzelmännchen,
das gerne weiterhilft
oder auch verwirrt *ggg*
Seitenanfangvoriger Beitragnächster BeitragSeitenende Link zu diesem Beitrag

Zaph (zaph)
Suche alle Beiträge dieser Person in dieser Hauptrubrik
Senior Mitglied
Benutzername: zaph

Nummer des Beitrags: 1264
Registriert: 07-2000
Veröffentlicht am Donnerstag, den 01. August, 2002 - 22:41:   Beitrag drucken

@Jan: Ausgezeichnetes Beispiel!!

Die Umformung von Gleichung (I und II) nach Gleichung (I - II) ist nicht äquivalent.

Ich denke, jetzt ist wirklich alles klar :-)

Gruß

Z.
Seitenanfangvoriger Beitragnächster BeitragSeitenende Link zu diesem Beitrag

Walter H. (mainziman)
Suche alle Beiträge dieser Person in dieser Hauptrubrik
Erfahrenes Mitglied
Benutzername: mainziman

Nummer des Beitrags: 101
Registriert: 05-2002
Veröffentlicht am Freitag, den 02. August, 2002 - 05:18:   Beitrag drucken

Hi Zaph,

Die Umformung von Gleichung (I und II) nach Gleichung (I - II) ist sehr wohl äquivalent.

Hat nur den Pferdefuß: "Man darf nicht folgern:
A => B; wenn B wahr ist, auch A wahr sein muß";

Auf gut Deutsch, ist die Annahme bereits falsch, ist es egal was rauskommt, weil nicht von einem richtigen Ergebnis auf eine richtige Annahme geschlossen werden darf.

Anderes Beispiel: aus den komplexen Zahlen

j^2 = -1

Annahme: j < 0

j < 0 | ^2 auf beiden Seiten
j^2 < 0
-1 < 0 | das ist zwar richtig,
aber die Annahme war bereits falsch; und daher
kann nicht auf eine richtige Annahme geschlossen
werden.

Gruß,
Walter

Mainzi Man,
ein Mainzelmännchen,
das gerne weiterhilft
oder auch verwirrt *ggg*
Seitenanfangvoriger Beitragnächster BeitragSeitenende Link zu diesem Beitrag

clara
Suche alle Beiträge dieser Person in dieser Hauptrubrik
Unregistrierter Gast
Veröffentlicht am Freitag, den 02. August, 2002 - 12:08:   Beitrag drucken

Hi Walter,
die Annahme macht doch wohl eher keinen Sinn, weil die komplexen Zahlen nicht angeordnet sind.

Und ich bin auch der Meinung, dass die Umformungen von Robert nicht äquivalent sind. Wenn man die Bedeutung von äquivalenten Umformungen auf Gleichungen bezieht, dann geht es darum lineare Gleichungssysteme zu lösen. Also Gleichungen mit Unbekannten und addieren, subtrahieren etc. bedeutet dann immer nur, dass das entstehende lineare Gleichungssystem dieselbe Lösungsmenge hat, wie das vorherige. In diesem Fall geht es aber gar nicht um das lösen eines Gleichungssystems.
Ein anderes allgemeines Beispiel:

I. a = b , dann folgt daraus b = a
II. b = a
beide addieren liefert: a + b = b + a.
Das ist eine wahre Aussage, aber du willst doch wohl nicht behauptet, dass ich jetzt bewiesen habe, dass alle Zahlen gleich sind?
Von der Gleichung a + b = b + a kommt man nicht zur Gleichung a = b. Nur wenn man benutzt, dass a = b ist.
gruß clara
Seitenanfangvoriger Beitragnächster BeitragSeitenende Link zu diesem Beitrag

Robert (emperor2002)
Suche alle Beiträge dieser Person in dieser Hauptrubrik
Mitglied
Benutzername: emperor2002

Nummer des Beitrags: 32
Registriert: 04-2002
Veröffentlicht am Freitag, den 02. August, 2002 - 13:32:   Beitrag drucken

Hi Clara!

Danke nochmals für dein Beispiel, aber mittlerweile habe auch ich es aufgegeben und es eingesehen, was falsch ist :-)

frosch

Gruß
MFG Robert

www.mathefreak.de / webmaster@mathefreak.de
Seitenanfangvoriger Beitragnächster BeitragSeitenende Link zu diesem Beitrag

Walter H. (mainziman)
Suche alle Beiträge dieser Person in dieser Hauptrubrik
Erfahrenes Mitglied
Benutzername: mainziman

Nummer des Beitrags: 102
Registriert: 05-2002
Veröffentlicht am Freitag, den 02. August, 2002 - 18:03:   Beitrag drucken

Hallo Clara,

mir gings darum anzumerken, daß
man eine falsche/unsinnige Annahme sehr wohl durch
äquivalente Umformungen in eine richtige/sinnvolle
Aussage bringen kann, aber da nicht daruf schließen darf, daß die Annahme sinnvoll/richtig ist.

Dann und nur dann, wenn die Annahme richtig/sinnvoll ist, kann man mit einer äquivalenten Umformung eine Aussage bekommen von der geschlossen werden kann, daß die Annahme sinnvoll/richtig ist.

Gruß,
Walter

p.s. kann es sein, daß Du mein posting nicht verstanden hast.
Mainzi Man,
ein Mainzelmännchen,
das gerne weiterhilft
oder auch verwirrt *ggg*
Seitenanfangvoriger Beitragnächster BeitragSeitenende Link zu diesem Beitrag

Christian Schmidt (christian_s)
Suche alle Beiträge dieser Person in dieser Hauptrubrik
Mitglied
Benutzername: christian_s

Nummer des Beitrags: 44
Registriert: 02-2002
Veröffentlicht am Freitag, den 02. August, 2002 - 18:37:   Beitrag drucken

Der Thread is ja durch meinen kleinen Einwand ziemlich lang geworden mittlerweile ;)

Seitenanfangvoriger Beitragnächster BeitragSeitenende Link zu diesem Beitrag

Ingo (ingo)
Suche alle Beiträge dieser Person in dieser Hauptrubrik
Moderator
Benutzername: ingo

Nummer des Beitrags: 488
Registriert: 08-1999
Veröffentlicht am Samstag, den 03. August, 2002 - 11:03:   Beitrag drucken

Das ist schlichtweg falsch, Walter.
Äquivalente Umformungen zeichnet ja gerade aus, daß sie in beide Richtungen gemacht werden können. Was Du meinst sind Implikationen.

Und daß die Implikation falsch=>wahr durchaus richtig ist, haben ja schon ein paar Leute hier beschrieben.

Du hast zum Beispiel aus j<0 gefolgert, daß j²<0 (was ich sowieso nicht einsehe, da es den Anordnungsaxiomen wiederspricht. Aus j<0 folgt j*j>0). Quadrieren ist aber nur dann eine Äquivalenzumformung wenn es auf IR+ oder IR- vorgenommen wird. Folglich handelt es sich bei deinem Beispiel eben nicht um Äquivalenzen.

Und um nochmal auf den Beweis von Robert einzugehen:
Äquivalent sind in der Tat x=0,999.. und 10x=9,999.. Aber daß 9x=10x-1x=9 gilt, ist nicht äquivalent zu x=0,999... denn dividieren durch 9 liefert x=9/9=1 und daß das gerade 0,99.. entspricht, soll ja erst bewiesen werden.
Seitenanfangvoriger Beitragnächster BeitragSeitenende Link zu diesem Beitrag

Walter H. (mainziman)
Suche alle Beiträge dieser Person in dieser Hauptrubrik
Erfahrenes Mitglied
Benutzername: mainziman

Nummer des Beitrags: 103
Registriert: 05-2002
Veröffentlicht am Samstag, den 03. August, 2002 - 14:46:   Beitrag drucken

Hi Ingo,

man spricht, wenn man auf beiden Seiten der Gleichung das gleiche macht immer von Äquivalenzumformungen;
Was Du meinst ist was anderes;

Und genau das ist der Grund, warum zwischenschritte angegeben werden müssen;

Oder wie meinst Du ist folgendes zu interpretieren?
a = b + 1 <=> a = 7
Ohne die Angabe der Zwischenschritte gar nicht;

Klar?

Gruß,
Walter
Mainzi Man,
ein Mainzelmännchen,
das gerne weiterhilft
oder auch verwirrt *ggg*
Seitenanfangvoriger Beitragnächster BeitragSeitenende Link zu diesem Beitrag

clara
Suche alle Beiträge dieser Person in dieser Hauptrubrik
Unregistrierter Gast
Veröffentlicht am Samstag, den 03. August, 2002 - 15:53:   Beitrag drucken

@Walter,
langsam denke ich, dass wir einfach eine andere Definition von Äquivalenzumformung benutzen. Also:
Wie ich schon mal vorher geschrieben habe bedeutet es bei Gleichungen, dass die Lösungsmenge der einen Gleichung dieselbe ist, wie die der anderen und umgekehrt.
Auf beiden Seiten dasselbe tun ist dafür eine nicht korrekte Beschreibung, weil z.B. quadrieren KEINE Äquivalenzumformung ist.
Wurzel(x) = -1 => x^2 = 1 => x = 1 oder x = -1,
ist richtig, aber das sind Implikationen. Die Äquivalenz gilt nicht, weil
x^2 = 1 => Wurzel(x) = -1 falsch ist.
Man muß wirklich genau aufpassen. Es ist ja auch z.B.
cos(x) = 0 => x = 90grad falsch und
cos(x) = 0 <= x = 90grad richtig.

Für gewöhnlich (ich habe es anders noch nie gesehen) werden Äquivalenzumformungen von Gleichungen so definiert wie oben und dann gibt es Sätze darüber, was Äquivalenzumformungen sind, nämlich das addieren und subtrahieren gleicher Zahlen auf beiden Seiten und das multiplizieren bzw. dividieren gleicher Zahlen ungleich null.

gruß clara
Seitenanfangvoriger Beitragnächster BeitragSeitenende Link zu diesem Beitrag

Ingo (ingo)
Suche alle Beiträge dieser Person in dieser Hauptrubrik
Moderator
Benutzername: ingo

Nummer des Beitrags: 491
Registriert: 08-1999
Veröffentlicht am Samstag, den 03. August, 2002 - 17:33:   Beitrag drucken

@Walter

Ich stimme clara zu. Der Begriff der Äquivalenz ist meines Wissens mathematisch eindeutig definiert. Was Du als Äquivalenz beschreibst ist einfach nur eine Implikation,oder auch Folgerung.
Mich würde interessieren, wer Dir diesen falschen Begriff beigebracht hat. Hab nämlich eben nochmal das Internet durchforstet und überall ähnliche Definitionen gefunden, die sich nicht mit deiner Aussage decken.

Beispiele:
Mathe Online
Eine Äquivalenzumformung besteht darin, die linke und die rechte Seite einer Gleichung auf gleiche Weise abzuändern. Diese Änderung muß allerdings umkehrbar sein: es muß möglich sein, die ursprüngliche Gleichung durch eine weitere Umformung zurückzugewinnen

Uni Bayreuth
Gleichungen bzw. Ungleichungen mit demselben Grundbereich, die die gleiche Lösungsmenge haben, heißen zueinander äquivalent. Eine Umformung, mit der man von einer Gleichung zu einer (zu dieser) äquivalenten Gleichung gelangt, heißt Äquivalenzumformung.

Verein
Mathematik-Olympiade-Siegerland e.V.

Seien A und B zwei Aussagen. A heißt äquivalent zu B genau dann, wenn sie den gleichen Wahrheitswert besitzen

Seitenanfangvoriger Beitragnächster BeitragSeitenende Link zu diesem Beitrag

Zaph (zaph)
Suche alle Beiträge dieser Person in dieser Hauptrubrik
Senior Mitglied
Benutzername: zaph

Nummer des Beitrags: 1274
Registriert: 07-2000
Veröffentlicht am Samstag, den 03. August, 2002 - 17:38:   Beitrag drucken

Ist das in Österreich vielleicht mal wieder anders? ;-)
Seitenanfangvoriger Beitragnächster BeitragSeitenende Link zu diesem Beitrag

Fanny
Suche alle Beiträge dieser Person in dieser Hauptrubrik
Unregistrierter Gast
Veröffentlicht am Samstag, den 03. August, 2002 - 19:50:   Beitrag drucken

Es ist zwecklos den Walter überzeugen zu wollen. Er hat schlicht von Mathe keine Ahnung! Er weiß ja nicht einmal was eine Seitenhalbierende in einem Dreieck ist!
Seitenanfangvoriger Beitragnächster BeitragSeitenende Link zu diesem Beitrag

Robert (emperor2002)
Suche alle Beiträge dieser Person in dieser Hauptrubrik
Mitglied
Benutzername: emperor2002

Nummer des Beitrags: 41
Registriert: 04-2002
Veröffentlicht am Samstag, den 03. August, 2002 - 20:28:   Beitrag drucken

@Fanny

Ich bitte doch um mehr toleranz. Vielleicht hättest du dir des Posting mit der Seitenhalbierenden mal GENAU und bis zum Ende durchgelesen.

Denn Seitenhalbierende hat er nicht gelernt, er hat einen anderen Begriff gelernt. Und seine Diskussion damals ging um die SEMANTIK (Wortsinn) des Begriffes Seitenhalbierende!!!

Also ein bisschen mehr Zurückhaltung!!!

Gruß Robert
MFG Robert

www.mathefreak.de / webmaster@mathefreak.de
Seitenanfangvoriger Beitragnächster BeitragSeitenende Link zu diesem Beitrag

Walter H. (mainziman)
Suche alle Beiträge dieser Person in dieser Hauptrubrik
Erfahrenes Mitglied
Benutzername: mainziman

Nummer des Beitrags: 104
Registriert: 05-2002
Veröffentlicht am Samstag, den 03. August, 2002 - 20:41:   Beitrag drucken

Jetzt tät es mich aber interessieren, wenn da immer davon gesprochen wird, daß Addition und Subtraktion als Äquivalenzumformungen gelten, warum jeder Robert mit dem Beweis, daß 0,p9 = 1 dermassen kritisiert => das sind äquivalente Umformungen;

@clara: ich hatte mich nicht klar ausgedrückt, aber bei einer Gleichung auf beiden Seiten den gleichen Term zu addieren bzw. zu subtrahieren oder mit der gleichen Konstante bzw. durch die gleiche Konstante zu dividieren ist eine Äquivalente Umformung;

Und das erheben in eine höhere Potenz (hier nur nat.) ist genau dann eine Äquivalenzumformung wenn n eine ungerade Zahl oder n gerade und auf beiden Seiten das gleiche Vorzeichen steht;

sqrt( x - 7 ) = x - 10
hier darf nur quadriert werden wenn x >= 10 ist
obwohl die Wurzel auch für Werte 7 <= x < 10 definiert ist;
x - 7 = x^2 - 20x + 100
x^2 - 21x + 107 = 0
x1,2 = 21/2 +/- sqrt( 441/4 - 107 )
x1,2 = 21/2 +/- sqrt(13)/2
=> hier darf nur die Lösung mit + genommen werden
weil die mit - nicht unsere Bedingung erfüllt.

Klar wie das ganze gemeint war?

@Ingo: mir ist klar worums geht; ich versteh nur nicht warum keiner mein Posting 2. Aug. 2002, 6:18 vollständig gelesen hat;

@Zaph: Warum hast Du dann den Robert niedergemacht, der nur äquivalente Umformungen gemacht hat;

@Fanny: hauptsache Du weißt was a Seitensymmetrale ist;

@Alle: wenn die Definition einer Äquivalenzumformung, welche ich angeblich falsch verwende, so lautet ist die Beweismethode der vollst. Induktion aber sehr sehr in Frage zu stellen.

Gruß,
Walter
Mainzi Man,
ein Mainzelmännchen,
das gerne weiterhilft
oder auch verwirrt *ggg*
Seitenanfangvoriger Beitragnächster BeitragSeitenende Link zu diesem Beitrag

clara
Suche alle Beiträge dieser Person in dieser Hauptrubrik
Unregistrierter Gast
Veröffentlicht am Montag, den 05. August, 2002 - 10:29:   Beitrag drucken

@Walter
letzter Versuch:
Zumindestens bin ich jetzt soweit, dass ich weiß, dass wir beide dieselbe Definition von Äquivalenzumformungen im Kopf haben.
Das was bei Robert falsch ist, ist folgendes:
Wenn er aus seiner Gleichung I. I'. macht ist es noch o.k., aber wenn er sie dann addiert, dann addiert er nur dann auf beiden Seiten dieselbe Zahl, wenn seine Gleichung I. wahr ist, aber das will er ja gerade zeigen und deswegen ist es keine Äquivalenzumformung.

Und was hat das nun mit der vollständigen Induktion zu tun? Das verstehe ich nicht.

gruß clara
Seitenanfangvoriger Beitragnächster BeitragSeitenende Link zu diesem Beitrag

Pesten
Suche alle Beiträge dieser Person in dieser Hauptrubrik
Unregistrierter Gast
Veröffentlicht am Dienstag, den 06. August, 2002 - 15:04:   Beitrag drucken

Hi Leute!

Ne lustig Diskussion, die Ihr hier führt.
Auf die gleiche Frage wie Missi kamen wir damals in der Schule auch (0.p9=1?)
0.p9 lässt sich aus keinem gemeinen Bruch bilden. Es sei denn man geht einen Umweg:

0.p8+0.p1=0.p9

8/9+1/9=0.p9
9/9 =0.p9
1 =0.p9 q.e.d. (ohne Gleichungen zu subtrahieren oder andere "komplizierte" Äquivalenzumformungen *gg*)

Caio Pesten
Seitenanfangvoriger Beitragnächster BeitragSeitenende Link zu diesem Beitrag

Walter H. (mainziman)
Suche alle Beiträge dieser Person in dieser Hauptrubrik
Erfahrenes Mitglied
Benutzername: mainziman

Nummer des Beitrags: 112
Registriert: 05-2002
Veröffentlicht am Dienstag, den 06. August, 2002 - 18:14:   Beitrag drucken

Hi Pesten,

Dein Beweis hat eine große Lücke.

Beweis: 0.p8 = 8/9 fehlt
Beweis: 0.p1 = 1/9 fehlt
weil aus dem automatisch folgen 0.p9 = 9/9 = 1 würde, was aber gezeigt werden muß.
klar?

Gruß,
Walter

@clara: soweit ich mich erinnere geht man z.B. bei Ungleichungen welche man mittels vollst. Induktion beweist in etwa so vor:
Tl(x) < Tr(x)
Ind. Voraus: x = 1, ok; x = 2, ok;
Ind. Schluss: Tl(x+1) < Tr(x+1)
dann macht man meist das:
Tl(x+1) - Tl(x) < Tr(x+1) - Tr(x)
oder
Tl(x+1) / Tl(x) < Tr(x+1) / Tr(x)
und zeigt dies relativ einfach, weil Tl(x+1) fast alle Termglieder/Faktoren von Tl(x) bzw. Tr(x+1) fast alle Termglieder/Faktoren von Tr(x) enthält;
Und dass da Äquivalent sein muß is doch ohne Zweifel, oder?

Mainzi Man,
ein Mainzelmännchen,
das gerne weiterhilft
oder auch verwirrt *ggg*
Seitenanfangvoriger Beitragnächster BeitragSeitenende Link zu diesem Beitrag

Raphael
Suche alle Beiträge dieser Person in dieser Hauptrubrik
Unregistrierter Gast
Veröffentlicht am Dienstag, den 06. August, 2002 - 20:30:   Beitrag drucken

Zu Vorwürfe gegen Walter:
Ich kann nicht verstehen, dass einige Leute(von denen man meist nichts hört)
sofort behaupten "der und der" hat von Mathematik keine Ahnung, weil "der und der" vielleicht(nur vielleicht) mal einen Fehler gemacht hat. Hier haben schon wirkliche Korypheren Fehler gemacht(Gott sei Dank sind Sie auch bloss Menschen!),das ist ganz normal.
Mich würde interessieren wie viel Ahnung die Leute haben, die sagen "Walter hat keine Ahnung," denn aus der Aussage "Walter hat keine Ahnung von Seitenhalbierenden" folgt nicht :
Walter hat keine Ahnung von Mathematik.
Ich bin Überzeugt, dass kompetentere Leute Walters Kompetenz schriftlich bestätigt haben!
Seitenanfangvoriger Beitragnächster BeitragSeitenende Link zu diesem Beitrag

Purzelbaum
Suche alle Beiträge dieser Person in dieser Hauptrubrik
Unregistrierter Gast
Veröffentlicht am Dienstag, den 06. August, 2002 - 23:03:   Beitrag drucken

Schlussfolgerung:
auch wenn man nicht weiß was Äquivalenzumformungen und was Seitenhalbierende sind *), kann man ein guter Mathematiker sein!

*) und es auch nach Erklärungsversuchen nicht versteht
Seitenanfangvoriger Beitragnächster BeitragSeitenende Link zu diesem Beitrag

Pesten
Suche alle Beiträge dieser Person in dieser Hauptrubrik
Unregistrierter Gast
Veröffentlicht am Mittwoch, den 07. August, 2002 - 09:35:   Beitrag drucken

@Walter

muss ich beweisen, dass 1/9=0.p1?
muss ich beweisen, dass 1/2=0.5?

Deshalb nix klar...

Pesten
Seitenanfangvoriger Beitragnächster BeitragSeitenende Link zu diesem Beitrag

Walter H. (mainziman)
Suche alle Beiträge dieser Person in dieser Hauptrubrik
Erfahrenes Mitglied
Benutzername: mainziman

Nummer des Beitrags: 113
Registriert: 05-2002
Veröffentlicht am Mittwoch, den 07. August, 2002 - 12:32:   Beitrag drucken

@Pesten

Du mußt beweisen dass 0.p1 = 1/9
weil ja da automatisch folgt,
dass 0.p9 = 9/9 = 1 und genau das willst ja beweisen, oder?

Klar?

Gruß,
Walter
Mainzi Man,
ein Mainzelmännchen,
das gerne weiterhilft
oder auch verwirrt *ggg*
Seitenanfangvoriger Beitragnächster BeitragSeitenende Link zu diesem Beitrag

clara
Suche alle Beiträge dieser Person in dieser Hauptrubrik
Unregistrierter Gast
Veröffentlicht am Mittwoch, den 07. August, 2002 - 13:17:   Beitrag drucken

Hi,
nun bin ich platt. Dass 1/9 = 0,p1 ist kann man doch einfach schriftlich ausrechnen.??????? Wo ist denn da mein Denkfehler?

Das mit der Induktion verstehe ich immer noch nicht.
Das was bei dir die I-Vor. ist, ist bei mir immer der I-Anfang und die I-Vor ist bei mir, dass die Behauptung wahr ist für ein x aus N bzw. für alle die kleiner oder gleich x sind und dann ist zu zeigen, dass es auch für x+1 wahr ist und man darf benutzen, dass es für alle kleineren wahr ist.

clara
Seitenanfangvoriger Beitragnächster BeitragSeitenende Link zu diesem Beitrag

Pesten
Suche alle Beiträge dieser Person in dieser Hauptrubrik
Unregistrierter Gast
Veröffentlicht am Mittwoch, den 07. August, 2002 - 14:03:   Beitrag drucken

und schon hab ich jemanden, der auch der Meinung ist, dass man nicht beweisen braucht, was mann nachrechnen kann.

DANKE CLARA

@Walter

Es sollte weniger ein Beweis werden, als zu zeigen, dass man auch auf anderem Wege zu der Aussage 0.p9=9/9 kommt.

Aber wenn ich noch beweisen soll, dass Wasser nass ist, dann bitte (mit Deinen Mitteln):

x = 0,p1
<=>
10*x = 1,p1
beide äquivalenten Gleichungen subtrahieren
9*x = 1
<=>
x = 1/9

Vorausgesetzt das Verfahren hat keine Fehler, passt das ja wohl.

Ciao Pesten
Seitenanfangvoriger Beitragnächster BeitragSeitenende Link zu diesem Beitrag

ich
Suche alle Beiträge dieser Person in dieser Hauptrubrik
Unregistrierter Gast
Veröffentlicht am Mittwoch, den 07. August, 2002 - 14:12:   Beitrag drucken

...nicht nur mann - auch frau kann rechnen!!
Seitenanfangvoriger Beitragnächster BeitragSeitenende Link zu diesem Beitrag

Walter H. (mainziman)
Suche alle Beiträge dieser Person in dieser Hauptrubrik
Erfahrenes Mitglied
Benutzername: mainziman

Nummer des Beitrags: 114
Registriert: 05-2002
Veröffentlicht am Mittwoch, den 07. August, 2002 - 14:33:   Beitrag drucken

@clara, das ist mir bekannt, daß man das einfach ausrechnen kann;

1/9 = 0,p1 | beide seiten mal 9
9/9 = 0,p9
1 = 0,p9
quod erat demonstrandum

und genau sowas ähnliches, wenn ich mich nicht verlesen hab, hat Robert gemacht, aber dem wurde es ausgebuht, warum?
Da bin ich jetzt platt;




siehe mein Posting vom
Donnerstag, den 01. August, 2002 - 22:20

Gruß,
Walter
p.s. eine Rechnung hängt doch nicht davon ab, wer sie macht, oder?

Mainzi Man,
ein Mainzelmännchen,
das gerne weiterhilft
oder auch verwirrt *ggg*
Seitenanfangvoriger Beitragnächster BeitragSeitenende Link zu diesem Beitrag

clara
Suche alle Beiträge dieser Person in dieser Hauptrubrik
Unregistrierter Gast
Veröffentlicht am Mittwoch, den 07. August, 2002 - 18:20:   Beitrag drucken

Hi Walter,
erst mal etwas allgemeines, da man aus dem Geschriebenen keine Emotionen lesen kann.
Ich meine nichts böse oder will Dir irgendwie Inkompetenz unterstellen. Ich bin lediglich daran interessiert eine Meinungsverschiedenheit zu beseitigen. Also: Sollte irgend etwas von meinem Geschriebenen anders angekommen sein, dann war es nicht so gemeint.

Natürlich ist eine Rechnung nicht davon abhängig wer sie macht, aber Robert hat in seiner Rechnung einen kleinen Teil anders gemacht und zwar, dass er Zahlen addiert hat, von denen er nicht wußte ob sie gleich sind und das war ja keine Äquivalenzumformung. In Deiner Rechnung oben hast Du das nicht gemacht und deswegen ist sie ein Beweis und das andere nicht.
Mathematik ist doch so genau.

Mit der Induktion bin ich nun immer noch nicht weiter.

gruß clara

P.S. Ich finde es schön, dass man mit Dir solche Dinge wirklich "ausschreiben" kann, ohne dass es oberflächlich wird bzw. gänzlich am Thema vorbei geht, wie man in einigen Beiträgen lesen kann.
Liegt vielleicht am Alter ;).
Seitenanfangvoriger Beitragnächster BeitragSeitenende Link zu diesem Beitrag

Zaph (zaph)
Suche alle Beiträge dieser Person in dieser Hauptrubrik
Senior Mitglied
Benutzername: zaph

Nummer des Beitrags: 1295
Registriert: 07-2000
Veröffentlicht am Mittwoch, den 07. August, 2002 - 23:09:   Beitrag drucken

Ich glaub es einfach nicht!

Walter, du beschwerst dich, dass wir angeblich deine (oder Roberts) Beiträge nicht richtig lesen. Tust du uns bitte den Gefallen, unsere Beiträge mal mit Verstand zu lesen?

Gruß

Z.
Seitenanfangvoriger Beitragnächster BeitragSeitenende Link zu diesem Beitrag

Robert (emperor2002)
Suche alle Beiträge dieser Person in dieser Hauptrubrik
Fortgeschrittenes Mitglied
Benutzername: emperor2002

Nummer des Beitrags: 53
Registriert: 04-2002
Veröffentlicht am Donnerstag, den 08. August, 2002 - 01:20:   Beitrag drucken

@All

Also da ja immer noch heiß diskutiert wird:

Ich habe bereits eingesehen, das mein Beweis fehlerhaft ist, was Zaph unf die anderen mir ja gesagt haben => ürigens Danke dazu :-)

@Walter

Dein Beweis oben ist ANDERS als mein Beweis, deiner ist allerdings gültig, aber meiner Aufgrund der Addition der Gleichungen nicht!

Also ich denke das damit die Diskusion über meinen fehlerhaften Beweis beendet ist ;)

Gruß Robert
MFG Robert

www.mathefreak.de / webmaster@mathefreak.de
Seitenanfangvoriger Beitragnächster BeitragSeitenende Link zu diesem Beitrag

Walter H. (mainziman)
Suche alle Beiträge dieser Person in dieser Hauptrubrik
Erfahrenes Mitglied
Benutzername: mainziman

Nummer des Beitrags: 118
Registriert: 05-2002
Veröffentlicht am Donnerstag, den 08. August, 2002 - 05:22:   Beitrag drucken

@clara:

Zum alten Eisen g'hör i nu lang nit;

@zaph:

Der Verstand hört auf, sobald man etwas in einer Sprache richtiges umdefiniert und damit sprachlich betrachtet falsch macht;

das engl. Wort f. Seitenhalbierende = "bisecting line" is genau das Problem, weil das engl. Wort "bisect" auf dt. heißt: halbieren bzw. in zwei Teile zerschneiden und genau des is auch a Mittelsenkrechte (in Österreich wird dafür das Wort Seitensymmetrale verwendet is aber des gleiche), und damit is die Definition als falsch bewiesen, quod erat demonstrandum
(Quelle: ISBN 3-468-05124-7)

Gruß,
Walter
Mainzi Man,
ein Mainzelmännchen,
das gerne weiterhilft
oder auch verwirrt *ggg*
Seitenanfangvoriger Beitragnächster BeitragSeitenende Link zu diesem Beitrag

Fanny
Suche alle Beiträge dieser Person in dieser Hauptrubrik
Unregistrierter Gast
Veröffentlicht am Donnerstag, den 08. August, 2002 - 20:59:   Beitrag drucken

Der Walter begreift es ja noch immer nicht!
Hoffnungsloser Fall.
Er kann nicht mit Verstand lesen, denn er hat keinen.
Seitenanfangvoriger Beitragnächster BeitragSeitenende Link zu diesem Beitrag

Walter H. (mainziman)
Suche alle Beiträge dieser Person in dieser Hauptrubrik
Erfahrenes Mitglied
Benutzername: mainziman

Nummer des Beitrags: 123
Registriert: 05-2002
Veröffentlicht am Freitag, den 09. August, 2002 - 08:57:   Beitrag drucken

@Fanny: jetzt nur für Dich

Es gibt ein Naturgesetz: Die Bevölkerung wächst;
und der Gesamtverstand bleibt a Konstante;

=> da ich vor Dir erschaffen worden bin, hast Du weniger als keinen Verstand <=> Dir muß man einen geben damit Du keinen Hast.
Mainzi Man,
ein Mainzelmännchen,
das gerne weiterhilft
oder auch verwirrt *ggg*
Seitenanfangvoriger Beitragnächster BeitragSeitenende Link zu diesem Beitrag

Pesten
Suche alle Beiträge dieser Person in dieser Hauptrubrik
Unregistrierter Gast
Veröffentlicht am Freitag, den 09. August, 2002 - 09:15:   Beitrag drucken

Mathematiker sind lustig:

@Walter

Ein Mathematiker sieht zwei Leute in ein Haus gehen und drei wieder herauskommen.
Was denkt er sich dabei?

Einer muss wieder rein, damit das Haus wieder leer ist.

Gruß Pesten
Seitenanfangvoriger Beitragnächster BeitragSeitenende Link zu diesem Beitrag

Tyll (tyll)
Suche alle Beiträge dieser Person in dieser Hauptrubrik
Mitglied
Benutzername: tyll

Nummer des Beitrags: 29
Registriert: 10-2001
Veröffentlicht am Freitag, den 09. August, 2002 - 22:49:   Beitrag drucken

Oder....daß vor Beobachtungsbeginn schon einer drin war, und gemäß der Betrachtungen, daß die Zahl von Menschen ein Element der natürlichen Zahlen (mit Null) ist, kommt er sicher auch zu der o.g. Aussage.
Gruß
Tyll

Beitrag verfassen
Das Senden ist in diesem Themengebiet nicht unterstützt. Kontaktieren Sie den Diskussions-Moderator für weitere Informationen.

ad

Administration Administration Abmelden Abmelden   Previous Page Previous Page Next Page Next Page